Violación del número bariónico en el modelo estándar a nivel perturbativo y no perturbativo

Esta es una continuación de mi pregunta aquí .

La página 635 de este libro de Matthew Schwartz dice efectivamente que el m j B m 0 dónde j B m es la corriente bariónica, es decir, el número bariónico no se conserva a nivel cuántico. Sin embargo, se puede demostrar que este término es una derivada total y, por lo tanto, cualquier diagrama de Feynman con este vértice contendrá un factor pag m = 0 . Dado que la teoría de perturbaciones se basa en los diagramas de Feynman, dicho término no puede contribuir en ningún orden a la teoría de perturbaciones.

Sin embargo, el término m j B m 0 se puede calcular a partir de diagramas triangulares.

¿No significa que la violación del número bariónico es posible (o al menos calculable) incluso en el nivel perturbativo?

Si es un efecto no perturbativo, ¿por qué es computable usando el diagrama de Feynman?

Si entiendo correctamente el argumento de Schwartz, la anomalía se puede derivar de los diagramas de Feynman, pero la anomalía en sí misma no da lugar a nuevos diagramas de Feynman. ¿Está bien?

¿Se supone que lee la ecuación (3.70) en su referencia? ¿Es esa la teoría de la perturbación?

Respuestas (1)

La violación del número bariónico ocurre solo cuando

Δ norte = d 4 X 0 | A ^ ( X ) | 0 0 ,
dónde A ^ ( X ) es la anomalía del número bariónico; esta es una relación exacta ya que la anomalía quiral es exacta en un bucle.

El VVE de d 4 X A ^ ( X ) obviamente depende del vacío que elija, y el vacío puede ser "perturbador" (es decir, simplemente el vacío de Fock con una solución trivial para los campos de calibre) o "no perturbador" - el θ -vacío. Para el primer caso, el valor no perturbado de la integral es idénticamente cero, mientras que la integral de cualquier excitación perturbativa se desvanece de manera idéntica debido a la razón que ha dado en la pregunta. Sin embargo, para el segundo caso, el VEV es en general distinto de cero.